Answer to Question #110279 in Electric Circuits for Ben

Question #110279
17.Calculate (a) the value of the resistor Rx, such that the total power dissipated in the circuit is 2.5KW, and (b) the current flowing in each of the four resistors . R1=15ohms, R2=10ohms, R3=38ohms and R4=Rx.
Note that R1 and R2 are connected parallel and R3 and Rx are also separately connected in parallel. The resistors are connected to 250 V power supply. (Draw the circuit diagram)
1
Expert's answer
2020-04-20T10:17:34-0400



a) The current in the circuit is: "I = P\/V = 2500\/250 = 10" A. The voltage across the resistors 1 and 2 by tne Ohm's law is: "U_{12} = I\\cdot \\dfrac{R_1R_2}{R_1+R_2} = 10\\cdot \\dfrac{15\\cdot10}{15+10} = 60" V.

The voltage across the resistors 3 and 4 is:

"U_{34} = U - U_{12} = 90" V.

The resistance of the resistor "R_4" can be found from the Ohm's law:

"U_{34} = I\\cdot \\dfrac{R_3R_4}{R_3+R_4} = 10\\cdot \\dfrac{38\\cdot R_4}{R_4+38} =90"

"R_4 = 11.8" Ohm


b) The currents in the resistors:

"I_1 = U_{12}\/R_1 = 60\/15 = 4" A;

"I_2 = U_{12}\/R_2 = 60\/10 = 6" A;

"I_3 = U_{34}\/R_3 = 90\/38 \\approx2.37" A;

"I_4 = U_{34}\/R_4 = 90\/11.8 \\approx 7.63" A.


Answer. a) 11.8 Ohm b) 4A, 6A, 2.37A, 7.63A.



Need a fast expert's response?

Submit order

and get a quick answer at the best price

for any assignment or question with DETAILED EXPLANATIONS!

Comments

No comments. Be the first!

Leave a comment

LATEST TUTORIALS
New on Blog
APPROVED BY CLIENTS